Đến nội dung

Secrets In Inequalities VP nội dung

Có 298 mục bởi Secrets In Inequalities VP (Tìm giới hạn từ 30-03-2020)



Sắp theo                Sắp xếp  

#535550 Trong mặt phẳng cho n giác đều A1A2...An .

Đã gửi bởi Secrets In Inequalities VP on 30-11-2014 - 16:07 trong Tổ hợp và rời rạc

n lẻ phải chỉ ra cách làm chứ.

n chẵn. Mỗi đồng tiền ở đính $i$ gán cho số $i$ , nếu chuyển đến điỉnh nào ta lại đổi cho đồng đó số ấy.

Nếu làm được như vầy ta phải có $\frac{n(n+1)}{2}\equiv na(modn)$ vô lí với n chẵn.




#535546 Cho tứ giác ABCD

Đã gửi bởi Secrets In Inequalities VP on 30-11-2014 - 15:34 trong Hình học

Gọi $X,Y$ là giao của $EF$ với $AB$ và $CD$, $K$ là giao của $AB$ và $CD$

Ta có : $(KXAB)=(KYDC)=-1$ $=>KX.KM=KA.KB=KD.KC=KY.KN$ . suy ra tứ giác $XYNM$ nội tiếp

$=>IM.IN=IX.IY=IE^2$ ( Do $(EFYX)=-1$. đpcm




#535541 Trong mặt phẳng cho n giác đều A1A2...An .

Đã gửi bởi Secrets In Inequalities VP on 30-11-2014 - 15:18 trong Tổ hợp và rời rạc

n chẵn nên lấy được 2 đỉnh làm đường kính đa giác, mỗi lần lấy tiền ở 2 đỉnh đối xứng nhau qua đường kính đó.Suy ra có thể .

Sr,nhầm vs n lẻ,bài này bất biến bt thôi :D




#507979 $$P=(a^2+b^2-ab)(b^2+c^2-bc)(a^2+c^2-ac)$$

Đã gửi bởi Secrets In Inequalities VP on 20-06-2014 - 10:20 trong Bất đẳng thức và cực trị

Giả sử $a= min$

Suy ra $a^{2}+b^{2}-ab\leq b^{2};a^{2}+c^{2}-ac\leq c^{2}$

Do đó $P\leq b^2c^2(b^2-bc+c^2)= b^2c^2[(b+c)^2-3bc]\leq b^2c^2(3^2-3bc)$

Đến đây $AM-GM$ hoặc ksht là ra.




#507977 cmr tồn tại vô số cặp số nguyên tố $(p,q)$

Đã gửi bởi Secrets In Inequalities VP on 20-06-2014 - 10:12 trong Số học

Chứng minh rằng tồn tại vô số cặp số nguyên tố $(p,q)$ thỏa mãn

$\left\{\begin{matrix} p| 2^{q-1}-1\\q|2^{p-1}-1 \end{matrix}\right.$

Ta có 2 NX quen thuộc :

$NX1$ : Nếu số nguyên tố $p|2^{2^{n}}+1$ với $n>3$ thì $p-1 \vdots 2^{n+2}$

$NX2$ : $2^{2^{n}}+1\neq p^{k}$ với mọi $n>3$ có nghĩa là số này luôn có ít nhất 2 ước nguyên tố phân biệt.

Chọn $ p|2^{2^{n}}+1;q|2^{2^{n+1}}+1 $ suy ra $p-1\vdots 2^{n+2}$ và $q-1\vdots 2^{n+3}$ 

Khi đó ta có :

$2^{q-1}-1\vdots 2^{2^{n+3}}-1\vdots 2^{2^{n+1}}-1\vdots 2^{2^{n}}+1\vdots p$

$2^{p-1}-1\vdots 2^{2^{n+2}}-1\vdots 2^{2^{n+1}}+1\vdots q$

Vậy  chọn $p,q$ như trên là ta có đpcm.




#505154 Chứng minh rằng $n \not |...$

Đã gửi bởi Secrets In Inequalities VP on 09-06-2014 - 10:21 trong Số học

Cho $p$ là số nguyên tố, $n$ là số tự nhiên $>1$, chứng minh rằng :

$$n \not |1+p^{n-1}+p^{2(n-1)}+...+p^{(n-1)(p-1)}$$

$n$ nguyên tố thì sao thím

--------------
Thì vẫn thế :|




#505146 $a^{4}+b^{3}=c^{2}$

Đã gửi bởi Secrets In Inequalities VP on 09-06-2014 - 09:48 trong Số học

Chọn $a=\sqrt{3m^{2}+1}$ 

$b= m^{2}-1$

$c= m(m^2+3)$

Sau đó chọn cho nó tự nhiên .




#450588 Chứng minh rằng tồn tại $k$ số nguyên liên tiếp là số $square-...

Đã gửi bởi Secrets In Inequalities VP on 15-09-2013 - 11:14 trong Số học

Một số nguyên dương được gọi là số $square-free$ nếu nó là tích của các số nguyên tố phân biệt. Chứng minh rằng với mọi số $k\geq 1,k\in \mathbb{Z}$ cho trước thì luôn tồn tại $k$ số nguyên liên tiếp là số $square-free$.

Trong $9$ số tn liên tiếp có $1$ số chia hết cho $3^2$




#450043 Chứng minh rằng mỗi số nguyên không âm đều có thể được biểu diễn dưới dạng...

Đã gửi bởi Secrets In Inequalities VP on 13-09-2013 - 21:16 trong Số học

Chứng minh rằng mỗi số nguyên không âm đều có thể được biểu diễn dưới dạng $a^{2}+b^{2}-c^{2}$,với $a,b,c$ là các số nguyên dương a<b<c 

$2n= (3n)^2+(4n-1)^2-(5n-1)^2$

$2n+3= (3n+2)^2+(4n)^2-(5n+1)^2$




#450024 Tìm mọi nghiệm nguyên dương của phương trình:

Đã gửi bởi Secrets In Inequalities VP on 13-09-2013 - 20:47 trong Số học

Tìm mọi nghiệm nguyên dương của phương trình: 

$w^2+x^2+y^2=z^2$

Sử dụng bổ đề sau : Nếu $m, n,a, b, c, d$ là các số nguyên dương sao cho $m^2+n^2=(a^2+b^2)(c^2+d^2)$.

Thì tồn tại các số nguyên dương $a_1,b_1,c_1,d_1$ sao cho:
$a^2+b^2=a_1^2+b_1^2, c^2+d^2=c_1^2+d_1^2, m=a_1b_1+c_1d_1, n=a_1d_1-b_1c_1$




#448240 Chứng minh rằng : $m\vdots p$

Đã gửi bởi Secrets In Inequalities VP on 06-09-2013 - 18:38 trong Số học

CHo $p$ là số nguyên tố lẻ.Kí hiệu : ${S_a}= a+\frac{a^{2}}{2}+...+\frac{a^{p-1}}{p-1}$.

Giả sử ${S_3}+{S_4}-3{S_2}=\frac{m}{n}$. Chứng minh rằng : $m\vdots p$




#441824 $\frac{(np)!}{p^n.n!}$ nguyên dương

Đã gửi bởi Secrets In Inequalities VP on 10-08-2013 - 21:13 trong Số học

CM: $\frac{(np)!}{p^n.n!}$ là số nguyên dương 

$(np)! \equiv  (p!)^n.n! (mod p^{n+3})$

Với $p\geq 5$ xét đa thức $f(x)=(x-1)(x-2)...(x-p+1)$ sau đó cm $f(mp)\equiv f(p)(Mod p^{3})\forall m \in \mathbb{N}^{*}$ ta có đpcm.




#441358 Tôpic nhận đề Tổ hợp, rời rạc

Đã gửi bởi Secrets In Inequalities VP on 08-08-2013 - 20:44 trong Bài thi đang diễn ra

1.Họ và tên thật : Lê Minh Tuấn Anh
2.Đang học lớp 11A1, trường THPT Chuyên Vĩnh Phúc, Tỉnh Vĩnh Phúc.
3.Đề bài : CHo tập $X$ gồm $n$ phần tử. ${a_1},{a_2}...,{a_n}$ là các số thỏa mãn $\forall u\neq v \in X,\exists {a_i}: (u-{a_i})(v-{a_i})\leq 0$
Cmr : $k\geq [{log_3}n]$
4.Đáp án : 
Xét ánh xạ $f:u \in X \mapsto ({u_1},{u_2}...{u_k})$.Trong đó : ${u_i}=0,1,2$ khi $u< {a_i},u={a_i},u> {a_i}$
Dễ thấy $f$ đơn ánh.Mà số bộ tối đa là $3^k$.Suy ra $3^{k}\geq n\Rightarrow k\geq [{log_3}n]$ đpcm

 

 

 



#437087 $2^{\frac{n(n-1)}{2}}.(2^{n-1...

Đã gửi bởi Secrets In Inequalities VP on 22-07-2013 - 12:20 trong Số học

Chứng minh

$2^{\frac{n(n-1)}{2}}.(2^{n-1}-1)(2^{n-2}-1)...(2^2-1).(2^1-1)\vdots n$

Ta còn chứng minh được là $2^{\frac{n(n-1)}{2}}.(2^{n-1}-1)(2^{n-2}-1)...(2^2-1).(2^1-1)\vdots n!$ 




#431097 $2^{n}a+b|c^{n}+1$

Đã gửi bởi Secrets In Inequalities VP on 27-06-2013 - 19:56 trong Số học

Anh cũng đến đoạn này rồi nhưng chả thấy qen gì cả.Em thử nói tiếp xem nào ? :3




#430671 $2^{n}a+b|c^{n}+1$

Đã gửi bởi Secrets In Inequalities VP on 26-06-2013 - 09:37 trong Số học

Tìm tất cả các bộ số nguyên $(a,b,c)$ sao cho : $2^{n}a+b|c^{n}+1$ $\forall n\in\mathbb{N}^{*}$




#423027 Free-of-sum

Đã gửi bởi Secrets In Inequalities VP on 02-06-2013 - 07:55 trong Tổ hợp và rời rạc

 

Bài toán: Một tập các số thực sẽ được gọi là ''free-of-sums'' nếu tập đó không chứa bất kì các số 11f6ad8ec52a2984abaafd7c3b516503785c2072, 95cb0bfd2977c761298d9624e4b4d4c72a39974a, 395df8f7c51f007019cb30201c49e884b46b92fa (nguyên) nào với tính chất 8b242acfb9e4302bbd4704b38c1faf6bd82a63d8. Một "free-of-sums" con của e14f63325bccf1daf9ae73aac5f9080e83060ac4 gồm 13fbd79c3d390e5d6585a21e11ff5ec1970cff0c phần tử. Xác định giá trị lớn nhất có thể của 13fbd79c3d390e5d6585a21e11ff5ec1970cff0c.

 

Hình như là $n+1$




#422962 Chứng minh các ước nguyên tố của $n^{2}+n+1$ không lớn hơ...

Đã gửi bởi Secrets In Inequalities VP on 01-06-2013 - 22:08 trong Số học

Chứng minh rằng tồn tại vô số số nguyên dương n sao cho tất cả các ước nguyên tố của $n^{2}+n+1$ không lớn hơn $\sqrt{n}$

$n=k^{2^t}$ , $k>2$,$k$ chia 3 du 1




#415673 \[ \sqrt{2}\sqrt{k+a^2}\le \frac...

Đã gửi bởi Secrets In Inequalities VP on 30-04-2013 - 21:33 trong Số học

Cho $a\in \mathbb{Z}, n,k\in\mathbb{N}$ thỏa mãn $k\equiv -a^2\mod 2n$. Chứng minh rằng: 

\[ \sqrt{2}\sqrt{k+a^2}\le \frac{k+a^2}{2n}+n\le \frac{k+a^2}{2}+1 \]

Có phải chỉ ra dấu "=" không thế.Nếu k thì hình như hơi dễ ^^




#415345 Chứng minh rằng $2^{\phi (n)}-1$ có các ước số nguyê...

Đã gửi bởi Secrets In Inequalities VP on 29-04-2013 - 13:53 trong Số học

Bài toán: Cho $n$ là một số nguyên dương lẻ, $n\ge 5$ và có các ước số nguyên tố là $p_1,p_2,...,p_k$. Chứng minh rằng $2^{\phi (n)}-1$ có các ước số nguyên tố không thuộc tập ${p_1,p_2,...,p_k}$

Giả sử $n={p_1}^{s_1}.{p_2}^{s_2}...{p_k}^{s_k}$ suy ra $\phi (n)= \prod_{i=1}^{k}{p_i}^{s_i-1}({p_1}-1)$ suy ra $\phi (n)$ chẵn

$\Rightarrow \phi (n)= 2a\Rightarrow \prod_{i=1}^{k}{p_i}^{s_i-1}({p_1}-1)= 2a$

$\Rightarrow a\vdots ({p_i}-1)\forall i=1,2,...,k\Rightarrow 2^{a}-1\vdots 2^{P_i-1}-1\vdots {P_i}\forall i=1,2,...,k$

Lại có $2^{\phi (n)}-1= 2^{2a}-1= (2^{a}-1)(2^{a}+1)$.

Dễ thấy $gcd(2^{a}-1,2^{a}+1)= 1\Rightarrow gcd(2^a+1,{p_i})= 1\forall i=1,2...,k$ suy ra $2^a-1$ có  ước số nguyên tố không thuộc tập ${p_1,p_2,...,p_k}$

Suy ra $2^{\phi (n)}-1$ có  ước số nguyên tố không thuộc tập ${p_1,p_2,...,p_k}$.

-----------------

@ Đ : Hình như c chưa xét đến TH $n$ là số nguyên tố @@~

@ T.A : Bài t có liên quan gì đến $n$ là snt đâu @@~

@ T.A : mà đề nó cho $n$ có một đống ước nt r mà

@ Đ : Cái đoạn $\Rightarrow a\vdots ({p_i}-1)\forall i=1,2,...,k$ cần là hợp số. V~ cả 1 đống =))




#414819 Chứng minh rằng: Hình chữ nhật $mxn$ có thể phủ kín bằng các quân L...

Đã gửi bởi Secrets In Inequalities VP on 25-04-2013 - 21:19 trong Tổ hợp và rời rạc

Đề bài: Chứng minh rằng hình chữ nhật $mxn$ có thể phủ kín bằng các quân L-Tri-mi-nô khi và chỉ khi :

$mn\vdots 3$ ^ $\begin{bmatrix} mn \vdots 2,m,n>1 \\ (m-3)(n-3) \geq 12 \end{bmatrix}$

Quân L-Tri-mi-nô là quân thế nào bạn ?




#414796 $P(x)=1+\frac{x}{n+1}+\frac{x^2}...

Đã gửi bởi Secrets In Inequalities VP on 25-04-2013 - 20:31 trong Đa thức

Bài toán :

Ch0 số nguyên dương $n$ và số nguyên tố $p>n+1$. Chứng minh rằng đa thức $P(x)=1+\frac{x}{n+1}+\frac{x^2}{2n+1}+....+\frac{x^p}{pn+1}$ không có nghiệm nguyên.

Giả sử $P(x)$ có nghiệm nguyên $b$ .

$P(x)= 0\Leftrightarrow a_{p}x^p+a_{p-1}x^{p-1}+...+a_{2}x^2+a_{1}x+a_{0}=0$

trong đó ${a_i}= \frac{(n+1)(2n+1)...(pn+1)}{in+1}$

Do $p> n+1\rightarrow (p,n)=1\Rightarrow$ $A= {n+1,2n+1,...,pn+1}$ là hệ đầy đủ mod $p$.

$\Rightarrow$ có đúng một số $k$ sao cho $kn+1\vdots p;k\neq 1,0< kn+1< p^{2}\rightarrow kn+1\not\vdots p^{2}$

$\Rightarrow {a_k}\not\vdots p;{a_i}\vdots p ,{a_i}\not\vdots p^{2}$ với mọi $i\neq k$. (1)

Vì $b$ là nghiệm suy ra $a_{p}b^p+a_{p-1}b^{p-1}+...+a_{2}b^2+a_{1}b+a_{0}=0\vdots p\rightarrow {a_k}b^k\vdots p\rightarrow b^{k}\rightarrow b\vdots p$

$\Rightarrow {a_t}b^{t}\vdots p\forall t=1,2,...,p\Rightarrow {a_0}\vdots p^{2}$.Vô lí theo (1)

Vậy giả sử sai và ta có đpcm




#414790 $x^2+y^2+z^2-wp=0$

Đã gửi bởi Secrets In Inequalities VP on 25-04-2013 - 20:13 trong Số học

 

CMR: với mọi số nguyên tố cho trước $p$ thì thì tồn tại số tự nhiên $x,y,z,w$ thỏa $x^2+y^2+z^2-wp=0$ và $0<w<p$

 

 

+Nếu $p=2$ chọn ngay $x=0,y=z=w=1$ ta có đpcm

+Nếu $p>2$.CHọn $z=1$.Xét các tập $A={x^2}$, $B={-y^2-1}$ với $x$ và $y$ lấy giá trị trong tập $C$ từ $0$ đến $\frac{p-1}{2}$.

Dễ thấy nếu $a,b\in C$ thì $a^{2}\not\equiv b^{2} (mod p)$ bởi vì nếu ngc lại thì $(a-b)(a+b)\vdots p$ nhg cả hai số này đều nhỏ hơn $p$ nên vô lí.

$\Rightarrow$ các phần tử trong $A$ và $B$ có số dư khác nhau khi chia cho $p$.

Mà $|A |+| B |= p+1> p$ nên tồn tại $2$ phần tử $x,y$ thuộc $A$ và $B$ sao cho $$x^{2}\equiv -y^{2}-1(Modp)\Rightarrow x^2+y^2+1= wp$$

$\Rightarrow 0< w=\frac{x^2+y^2+1}{p}\leq \frac{2(\frac{p-1}{2})^2+1}{p}< p$

Vậy chọn $x,y,z,w$ như trên ta có đpcm.




#414786 $P(x)=(x-a_1)^{2}(x-a_2)^{2}...(x-a_n)^{2}...

Đã gửi bởi Secrets In Inequalities VP on 25-04-2013 - 19:59 trong Đa thức

Chứng minh rằng với mọi số nguyên $a_1,a_2,...,a_n$ đôi một khác nhau,thì đa thức:$P(x)=(x-a_1)^{2}(x-a_2)^{2}...(x-a_n)^{2}+1$ không thể biểu diễn thành tích của hai đa thức (bậc dương) với hệ số nguyên.

Phản chứng giả sử $P(x)=g(x).h(x)$ ,$g(x),h(x) \in R[x]$ , $degg(x)+degh(x)= 2n$,$deg g(x)\leq deg h(x)\Rightarrow degg(x)\leq n$

$\Rightarrow g({a_i}).h({a_i})=P({a_i})= 1$$\forall i=1,2,...,2013$

$\Rightarrow$ $g(x)$ và $h(x)$ cùng đồng nhất bằng $1$ hoặc $-1$.

Nếu có $i,j$ sao cho $g({a_i})= 1; g({a_j})= -1\Rightarrow g({a_i}).g({a_j})< 0\Rightarrow \exists {x_0}:g({x_0})= 0$

$\Rightarrow P(x)$ có nghiệm ${x_0}$ vô lí vì $P(x)$ luôn dương với mọi $x$.

Do đó chỉ có thể xảy ra $g({a_i})=h({a_i})=1$ hoặc $g({a_i})=h({a_i})=-1$. với mọi $i$ chạy từ $1$ đến $n$

Xét TH $g({a_i})=h({a_i})=1$ , cái kia cmtt.

Vì $g({a_i})=1$ nên đa thức $g(x)-1$ có $n$ nghiệm từ ${a_1}$ đến ${a_n}$ nên bậc của nó sẽ lớn hơn hay bằng $n$ 

$degg(x)= degh(x)= n$

$\Rightarrow g(x)-1= c(x-{a_1})(x-{a_2})...(x-{a_n})$ , $ h(x)-1= d(x-{a_1})(x-{a_2})...(x-{a_n})$.

Mà $P(x)= g(x).h(x)$ nên so sánh hệ số cao nhất $2$ vế ta có $cd=1$.Giả sử là $c=d=1$.

Lại do $P(x)= g(x).h(x)$ $\Rightarrow \prod_{i=1}^{n}(x-{a_i})^{2}= (\prod_{i=1}^{n}(x-{a_i})+1).(\prod_{i=1}^{n}(x-{a_i})+1)$

$\Leftrightarrow 2\prod_{i=1}^{n}(x-{a_i})= 0\forall x\in R$.Vô lí.

Vậy giả sử sai và ta có đpcm




#411748 $$\left|\sum^{n}_{i=1} \frac...

Đã gửi bởi Secrets In Inequalities VP on 10-04-2013 - 21:15 trong Bất đẳng thức - Cực trị

Bài toán 2.

Ch0 $A=\{a_1;a_2;...;a_n\}$ là tập gồm $n$ số nguyên dương sa0 ch0 với mỗi 2 tập con $B,C$ rời nhau của $A$ ta có $\sum_{x\in B} x\neq \sum_{x\in C} x$. Chứng minh bất đẳng thức :

$$\frac{1}{a_1}+\frac{1}{a_2}+....+\frac{1}{a_n}<2$$

$A=\{1;2;3;4\}$ ???

--------

@@~ có $1+4=2+3$ mà